Question

(a) (5 points.) Let W CW CW CW3 be distinct subspaces of R? True/False (Justify your answers): (i) Wo must be the zero subspa
0 0
Add a comment Improve this question Transcribed image text
Answer #1

0 Answer: Let woch, ciecos be detinct subspaces. of Rs (2) Dimension of $?983. so posible Misspae of PR have dimension 91,2,3( cauld be constfen as BEW, & Wz de not exactly R. It has dimensions 10, 310 morphe cooth R. It is time statement. D, Ew but

Add a comment
Know the answer?
Add Answer to:
(a) (5 points.) Let W CW CW CW3 be distinct subspaces of R? True/False (Justify your...
Your Answer:

Post as a guest

Your Name:

What's your source?

Earn Coins

Coins can be redeemed for fabulous gifts.

Not the answer you're looking for? Ask your own homework help question. Our experts will answer your question WITHIN MINUTES for Free.
Similar Homework Help Questions
  • Mark each statement as True or False and justify your answer. a) The columns of a...

    Mark each statement as True or False and justify your answer. a) The columns of a matrix A are linearly independent, if the equation Ax = 0 has the trivial solution. b) If vi, i = 1, ...,5, are in RS and V3 = 0, then {V1, V2, V3, V4, Vs} is linearly dependent. c) If vi, i = 1, 2, 3, are in R3, and if v3 is not a linear combination of vi and v2, then {V1, V2,...

  • 1) Determine if w is in the subspace spanned by v1, v2, v3 2) Are the...

    1) Determine if w is in the subspace spanned by v1, v2, v3 2) Are the vectors v1, v2, v3 linearly dependent or independent? justify your answer Question 2. (15 pts) Let vi=(-3 0 6)", v2= (-2 2 3]", V3= (0 - 6 37, and w= [1 11 9". (1). Determine if w is in the subspace spanned by V1, V2, V3. (2). Are the vectors V1, V2, V3 linearly dependent or independent? Justify your answer

  • (1 point) Are the following statements true or false? ? 1. If W = Span{V1, V2,...

    (1 point) Are the following statements true or false? ? 1. If W = Span{V1, V2, V3 }, and if {V1, V2, V3 } is an orthogonal set in W, then {V1, V2, V3 } is an orthonormal basis for W. ? 2. If x is not in a subspace W, projw(x) is not zero. then x ? 3. In a QR factorization, say A = QR (when A has linearly independent columns), the columns of Q form an orthonormal...

  • Question 2. (15 pts) Let Vi= (-3 0 6)", v2= (-2 2 3)", V3= [0 -...

    Question 2. (15 pts) Let Vi= (-3 0 6)", v2= (-2 2 3)", V3= [0 - 6 3)", and w= [1 14 9)? (1). Determine if w is in the subspace spanned by V1, V2, V3. (2). Are the vectors Vi, V2, V3 linearly dependent or independent? Justify your answer.

  • Problem 5: Let V and W be vector spaces and let B = {V1, V2, ...,...

    Problem 5: Let V and W be vector spaces and let B = {V1, V2, ..., Un} CV be a basis for V. Let L :V + W be a linear transformation, and let Ker L = {2 € V: L(x)=0}. (a) If Ker L = {0}, show that C = {L(v1), L(02), ..., L(vn) } CW is a linearly independent set in W. (b) If C = {L(01), L(V2),..., L(Un)} C W is a linearly independent set in W,...

  • Question 2. (15 pts) Let vi= (-3 0 6)", V2= (-2 2 317, V3= [0 -...

    Question 2. (15 pts) Let vi= (-3 0 6)", V2= (-2 2 317, V3= [0 - 6 3)", and w=(1 14 9) (1). Determine if w is in the subspace spanned by va, V2, V3. (2). Are the vectors V1, V2, V3 linearly dependent or independent? Justify your answer.

  • Q10 10 Points Please answer the below questions. Q10.1 4 Points Let m, n EN\{1}, V...

    Q10 10 Points Please answer the below questions. Q10.1 4 Points Let m, n EN\{1}, V be a vector space over R of dimension n and (v1,..., Vm) be an m tuple of V. (Select ALL that are TRUE) If m > n then (v1, ..., Vy) spans V. If (01,..., Vm) is linearly independent then m <n. (V1,..., Um) is linearly dependent if and only if for all i = 1,..., m we have that Vi Espan(v1,..., Vi-1, Vi+1,...,...

  • 15 points) Consider the following vectors in R3 0 0 2 V1 = 1 ; V2...

    15 points) Consider the following vectors in R3 0 0 2 V1 = 1 ; V2 = 3 ; V3 = 1] ; V4 = -1;V5 = 4 1 2 3 = a) Are V1, V2, V3, V4, V5 linearly independent? Explain. b) Let H (V1, V2, V3, V4, V5) be a 3 x 5 matrix, find (i) a basis of N(H) (ii) a basis of R(H) (iii) a basis of C(H) (iv) the rank of H (v) the nullity...

  • Let v1= [−3 0 6]T , v2= [−2 2 3]T , v3= [0 − 6 3]T...

    Let v1= [−3 0 6]T , v2= [−2 2 3]T , v3= [0 − 6 3]T , and w= [1 14 9]T . (1). Determine if w is in the subspace spanned by v1, v2, v3. (2). Are the vectors v1, v2, v3 linearly dependent or independent? Justify your answer.

  • Please answer me fully with the details. Thanks! True of False? Justify yo ur answer. —D т. If {ii, .., in} is a linear...

    Please answer me fully with the details. Thanks! True of False? Justify yo ur answer. —D т. If {ii, .., in} is a linearly independent subset of (1) Let V bea vector spacе, аnd let dim(V) V. then n < т. (2) Let V and W be vector spaces, and suppose that T : V -+ W is a linear transformation. If there are vectors i, 2, ..., Tj in V such that the vectors T(),T(T2),...,T(vj) span W, then the...

ADVERTISEMENT
Free Homework Help App
Download From Google Play
Scan Your Homework
to Get Instant Free Answers
Need Online Homework Help?
Ask a Question
Get Answers For Free
Most questions answered within 3 hours.
ADVERTISEMENT
ADVERTISEMENT
ADVERTISEMENT